Use non-breaking space (~) between 'Figure' and the figure number.
[course.git] / latex / problems / Serway_and_Jewett_8 / problem30.64.tex
1 \begin{problem*}{30.64}
2 Two coplanar and concentric circular loops of wire carry currents of
3 $I_1=5.00\U{A}$ and $I_2=3.00\U{A}$ in opposite directions as in
4 Figure~P30.64.  If $r_1=12.0\U{cm}$ and $r_2=9.00\U{cm}$, what
5 are \Part{a} the magnitude and \Part{b} the direction of the magnetic
6 field at the center of the two loops?  \Part{c} Let $r_1$ remain fixed
7 at $12.0\U{cm}$ and let $r_2$ be variable.  Determine the value of
8 $r_2$ such that the net field at the center of the loops is zero.
9 \begin{center}
10 \begin{asy}
11 import Mechanics;
12 import ElectroMag;
13 import Circ;
14
15 real u = 0.2cm;
16 real r1 = 12u;
17 real r2 = 9u;
18 real dr = 6pt;
19
20 draw(scale(r1)*unitcircle, line);
21 draw(scale(r2)*unitcircle, line);
22 Distance d1 = Distance((0,0), r1*dir(-45), "$r_1$");
23 d1.draw();
24 Distance d2 = Distance((0,0), r2*dir(45), "$r_2$");
25 d2.draw();
26
27 draw(arc((0,0), r1+dr, angle1=10, angle2=-10), CurrentPen, ArcArrow);
28 label("$I_1$", (r1+dr)*dir(0), E);
29 draw(arc((0,0), r2-dr, angle1=170, angle2=190), CurrentPen, ArcArrow);
30 label("$I_2$", (r2-dr)*dir(180), E);
31 \end{asy}
32 \end{center}
33 \end{problem*}
34
35 \begin{solution}
36 \Part{a}
37 We're interested in the magnetic field generated at the center of the
38 ring.  There is no path for an Amperian loop that takes advantage of
39 the problem's symmetry, so we'll use the Biot--Savart law directly.
40 Integrating around a single ring of radius $r$, the magnetic field at
41 the center is given by
42 \begin{align}
43   \vect{B} &= \oint_S \dd\vect{B}
44     = \oint_S \frac{\mu_0}{4\pi}\cdot\frac{I \dd\vect{s} \times \rhat}{r^2}
45     = \frac{\mu_0 I}{4\pi r^2} \oint_S |\dd\vect{s}|\cdot|\rhat|\cdot\sin(\theta)
46     = \frac{\mu_0 I}{4\pi r^2} \oint_S |\dd\vect{s}|
47     = \frac{\mu_0 I}{4\pi r^2} \cdot 2\pi r
48     = \frac{\mu_0 I}{2 r} \;,
49 \end{align}
50 where we took advantage of the following facts:
51 \begin{itemize}
52   \item The current $I$ and distance $r$ from the wire to the point we
53     care about are the same for every segment $\dd\vect{s}$ in the
54     loop, so we can pull these constants outside the integral.
55  \item The cross product of two vectors can be written
56    $\vect{A}\times\vect{B}=|\vect{A}|\cdot|\vect{B}|\cdot\sin(\theta)$.
57  \item \rhat is a unit vector ($|\rhat|=1$).
58  \item \rhat always points centerward and $\dd\vect{s}$ is always
59    tangent to the ring, so $\theta=90\dg$ and $\sin(\theta)=1$.
60  \item $\oint_S |\dd\vect{s}|$ is just the total length of the path
61    $S$, which is the circumfernce of the ring ($2\pi r$).
62 \end{itemize}
63 Using the right hand rule, it is clear that a counterclockwise current
64 will generate a magnetic field at the center of the ring which points
65 out of the page.
66
67 Now that we have a formula for the magnetic field at the center of the
68 ring, we can calculate the magnetic field at the center of the two
69 rings using superposition.  Letting \emph{out of the page} be the
70 positive direction,
71 \begin{equation}
72   B = \frac{\mu_0 (-I_1)}{2 r_1} + \frac{\mu_0 I_2}{2 r_2}
73     = -5.24\U{$\mu$T} \;, \label{eq:30.64.a}
74 \end{equation}
75 so the magnetic field has a magnitude of $\ans{5.24\U{$\mu$T}}$.
76
77 \Part{b}
78 Because we picked \emph{out of the page} as the positive direction,
79 the $-$ sign in \cref{eq:30.64.a} means that the magnetic field points
80 \ans{into the page}.
81
82 \Part{c}
83 If we allow $r_2$ to vary, we have no magnetic field when
84 \begin{align}
85   0 &= B = \frac{\mu_0 (-I_1)}{2 r_1} + \frac{\mu_0 I_2}{2 r_2} \\
86   \frac{\mu_0 I_1}{2 r_1} &= \frac{\mu_0 I_2}{2 r_2} \\
87   \frac{I_1}{r_1} &= \frac{I_2}{r_2} \\
88   r_2 &= \frac{I_2}{I_1} r_1 = \ans{7.20\U{cm}} \;.
89 \end{align}
90 \end{solution}